User avatar
 
geverett
Thanks Received: 79
Atticus Finch
Atticus Finch
 
Posts: 207
Joined: January 29th, 2011
 
This post thanked 1 time.
 
 

Q14 - Some critics claim that the power

by geverett Thu Aug 11, 2011 11:13 am

I got this wrong on my PT. I chose answer B, because I misinterpreted it to be something along the lines of "Assumes that the media gives equal time to the individual opinions presented."

It doesn't say this at all as the answer choice just makes a blanket statement about the wide range of opinions in general.

Answer choice C is the correct answer. Here's the diagram:

If the media purveyed--------It would be true that the power of
range of opinions -------->the media to impose opinions is
narrower than found-----------too great.
among consumers

The media does not------------------It is not true that the power
purvey range of opinions -------> of the media to impose
narrower than found among--------- opinions is too great
consumers

So you can see that it sets up a conditional statement in the premise, and then makes a conclusion that is a mistaken negation of that conditional statement.
User avatar
 
ManhattanPrepLSAT1
Thanks Received: 1909
Atticus Finch
Atticus Finch
 
Posts: 2851
Joined: October 07th, 2009
 
 
 

Re: Q14 - Some critics claim that the power

by ManhattanPrepLSAT1 Fri Aug 19, 2011 9:08 pm

geverett Wrote:So you can see that it sets up a conditional statement in the premise, and then makes a conclusion that is a mistaken negation of that conditional statement.

Exactly, though we don't call it a "mistaken negation" around here! --:), that lingo is someone else's! We simply call it a Negation.

But yes...

If the media purveyed a range of opinion narrower than that found among consumers of media, then the critics claim would be true. Does that mean that if the range of opinion is not narrower than that found among consumers of media that the claim is false? Of course not!

Nice work geverett!
 
schweizeroo
Thanks Received: 0
Vinny Gambini
Vinny Gambini
 
Posts: 2
Joined: January 11th, 2012
 
 
 

Re: Q14 - Some critics claim that the power

by schweizeroo Tue Aug 14, 2012 12:45 pm

Also, and correct me if I'm wrong, I believe that supplying a range of opinions and the level of exposure those opinions get are not equivalent. One can provide a full range of opinions without giving all the opinions front page exposure. So, this would be a lanuage shift which provides further reason to reject B.
 
shiqi0628
Thanks Received: 0
Forum Guests
 
Posts: 6
Joined: September 11th, 2013
 
 
 

Re: Q14 - Some critics claim that the power

by shiqi0628 Mon Oct 28, 2013 6:06 am

hi, sorry to bring the old thread up, but i chose B on this one i really think i nailed the test maker's tricks . :?
but i reckon the 'negation' means the causation,which means he falsily take the negation as a sign of causation, since in a lot of examples we see the correlation of
~A----~B
A------B
then A and B are causally related

the wider range of opinions cause greater power, so the flaw would be neglecting certain alternative interpretations, such as much exposure as the wider range of important opinions they need.
and "tankes for granted flaw" implies a false necessary assumption, if you negate it: the media did not give as much exposure that as they should to a wider tange of opinions, then "the meida power is too great" conclusion is weakened, is it not...? probably the great power of media requires both narrow opinion and adequate exposure..?
probably im just overanalyzing it, thanks in advance!
User avatar
 
tommywallach
Thanks Received: 468
Atticus Finch
Atticus Finch
 
Posts: 1041
Joined: August 11th, 2009
 
 
 

Re: Q14 - Some critics claim that the power

by tommywallach Wed Oct 30, 2013 11:28 pm

Hey Shiql,

There are so many spelling and grammar errors in this post, I'm afraid we can't make heads or tails of it! People get used to being very loose with these things online, but on the LSAT, it's always important to be very careful with your words, because everything is about the details of the writing!

Good luck moving forward!

-t
Tommy Wallach
Manhattan LSAT Instructor
twallach@manhattanprep.com
Image
 
contropositive
Thanks Received: 1
Atticus Finch
Atticus Finch
 
Posts: 105
Joined: February 01st, 2015
 
 
 

Re: Q14 - Some critics claim that the power

by contropositive Wed Sep 02, 2015 6:38 pm

I did not find it necessary to do a conditional chain for this argument. I did pick B during timed test, but during review I realized B might be wrong because the argument does not assume/take for granted what B is saying. In fact, it actually explicitly tells us "this assumption is untrue..." in the last sentence. Is that reasoning right?
I picked C during review because I realized the last sentence sums up the flaw.
 
roflcoptersoisoi
Thanks Received: 0
Atticus Finch
Atticus Finch
 
Posts: 165
Joined: April 30th, 2015
 
 
 

Re: Q14 - Some critics claim that the power

by roflcoptersoisoi Mon Aug 15, 2016 11:31 am

Presumes that since a sufficient assumption is not true that the conclusion itself cannot be true.

(A) The argument does not launch any personal attacks
(B) The argument doesn't presuppose or discuss how much exposure the media should give to a wide range of issues on important issues of the day.
(C) Bingo
(D) The author doesn't make an appeal to popular opinion
(E) The argument does not even discuss the desirability for having wide range of opinion on important issues of the day.